Math, asked by vincy1194, 3 months ago

The probability distribution functions which shows the number of hours (X) a student study during

lockdown period in a day, is given by (where C > 0)


X 0 1 2


P(X) 3C^3
4C – 10C^2
5C – 1


On the basis of above information answer the following questions:


(i) The correct equation for C is

(a) 3C3
– 10C2 + C – 2 = 0
(b) 3C3 + 10C2 + C – 2 = 0

(c) 3C3
– 10C2 + 9C – 2 = 0
(d) None of these


(ii) The value of C is

(a) 1
/2
(b) 1
/3

c) 1
/4

(d) 1
/6


(iii) P(X < 2) =

(a) 1
/2

(b) 1
/3

(c) 1
/4

(d) 1
/6



(iv) P(X = 1) =

(a) 2
/9

(b) 1
/9

(c) 2
/3

(d) 1
/3


(v) P(X ≥ 0) =

(a) 1
/2

(b) 1
/3

(c) 1
(d) 0​

Answers

Answered by MaheswariS
3

\underline{\textbf{Given:}}

\textsf{The probability function of X is}

\begin{array}{|c|c|c|c|}\cline{1-4}X&amp;0&amp;1&amp;2\\\cline{1-4}P(X)&amp;3C^3&amp;4C-10C^2&amp;5C-1\\\cline{1-4}\end{array}

\underline{\textbf{To find:}}

\textsf{(i) correct equation for C}

\textsf{(ii) The value of C}

\mathsf{(iii)\;P(X &lt; 2)}

\textsf{(iv) P(X=1)}

\textsf{(iv)\;P(X{\geq}0)}

\underline{\textbf{Solution:}}

\mathsf{(i)}

\textsf{Since P(X) is a probability function,}

\mathsf{\displaystyle\sum\limits^{x=2}_{x=0}\;P(X=x)=1}

\implies\mathsf{P(X=0)+P(X=1)+P(X=2)=1}

\implies\mathsf{3C^3+4C-10C^2+5C-1=1}

\implies\mathsf{3C^3-10C^2+9C-1=1}

\implies\boxed{\mathsf{3C^3-10C^2+9C-2=0}}

\therefore\mathsf{Option\;(c)\;is\;correct}

\mathsf{(ii)\;consider,\;3C^3-10C^2+9C-2=0}

\implies\mathsf{3C^3-3C^2-7C^2+7C+2C-2=0}

\implies\mathsf{3C^2(C-1)-7C(C-1)+2(C-1)=0}

\implies\mathsf{(3C^2-7C+2)(C-1)=0}

\implies\mathsf{(3C^2-6C-C+2)(C-1)=0}

\implies\mathsf{(3C(C-2)-1(C-2))(C-1)=0}

\implies\mathsf{(3C-1)(C-2)(C-1)=0}

\implies\mathsf{C=1,2,\dfrac{1}{3}}

\mathsf{But\;C=1,2\;are\;not\;suitable\;for\;P(X)}

\implies\boxed{\mathsf{C=\dfrac{1}{3}}}

\therefore\mathsf{Option\;(b)\;is\;correct}

\mathsf{(iii)\;P(X&lt;2)}

\mathsf{=P(X=0)+P(X=1)}

\mathsf{=3C^3+4C-10C^2}

\mathsf{=3\left(\dfrac{1}{27}\right)+4\left(\dfrac{1}{3}\right)-10\left(\dfrac{1}{9}\right)}

\mathsf{=\dfrac{1}{9}+\dfrac{4}{3}-\dfrac{10}{9}}

\mathsf{=\dfrac{1+12-10}{9}}

\mathsf{=\dfrac{3}{9}}

\mathsf{=\dfrac{1}{3}}

\implies\boxed{\mathsf{P(X\,&lt;\,2)=\dfrac{1}{3}}}

\therefore\mathsf{Option\;(b)\;is\;correct}

\mathsf{(iv)\;P(X=1)}

\mathsf{=4C-10C^2}

\mathsf{=4\left(\dfrac{1}{3}\right)-10\left(\dfrac{1}{9}\right)}

\mathsf{=\dfrac{4}{3}-\dfrac{10}{9}}

\mathsf{=\dfrac{12-10}{9}}

\mathsf{=\dfrac{2}{9}}

\implies\boxed{\mathsf{P(X=1)=\dfrac{2}{9}}}

\therefore\mathsf{Option\;(a)\;is\;correct}

\mathsf{(v)\;P(x\;\geq\;0)}

\mathsf{=P(X=0)+P(X=1)+P(X=2)}

\mathsf{=1}

\implies\boxed{\mathsf{P(x\;\geq\;0)=1}}

\therefore\mathsf{Option\;(c)\;is\;correct}

\underline{\textbf{Find more:}}

Let A and B be two events such that P(A)=0.6, P(B) = 0.2 and P(A/B)=0.5. Then P (A'/B')

equals

(a) 1/10

(b) 3/10

(C) 3/8

(d) 6/7​

https://brainly.in/question/14116335

Answered by barani79530
0

Step-by-step explanation:

6/7

please mark as best answer and thank me

Similar questions